log8-log4 ÷ log4-log2=





Answers

Answer 1
The answer is log(4)-1

Related Questions

The quotient of (x5 – 3x3 – 3x2 – 10x + 15) and a polynomial is (x2 – 5). What is the polynomial?

Answers

Answer:

x⁷ – 8x⁵ – 3x⁴ + 15x³ + 20x² + 10x – 75

Step-by-step explanation:

(x⁵ – 3x³ – 3x² – 10x + 15) ˣ (x² – 5) = x⁷ – 8x⁵ – 3x⁴ + 15x³ + 20x² + 10x – 75

someone help me now?

Answers

Answer:

answer of given question is 200 miles

for this answer give me brilliant tag

Answer:

200 miles

Step-by-step explanation:

this is really not difficult.

just solve the equation as it is already written there.

130 = 0.5m + 30

100 = 0.5m = m/2

m = 200 miles

that is all that is to it.

putting this in here as question costs way more time than just doing this.

Solve for x.

Help me please

Answers

Answer:

x = 24

Step-by-step explanation:

mathematically, in a cyclic quadrilateral, two opposite angles are supplementary

what this mean is that they add up to be 180

From what we have in the question, the two angles are supplementary and that means they add up to equal 180 degrees

thus, we have it that;

(4x + 9) + (3x + 3) = 180

4x + 3x + 9 + 3 = 180

7x + 12 = 180

7x = 180-12

7x = 168

x = 168/7

x = 24

The Cooking Club made some pies to sell during lunch to raise money for a field trip. The cafeteria helped by donating three pies to the club. Each pie was then cut into six pieces and sold. There were a total of 72 pieces to sell. How many pies did the club make?
Which equation can be used to solve this problem? Select all that apply.

Answers

Yes hhhgfduujhcffykkbvfgy Jajdud fdtiobds

The following table shows the amount of red paint that can be added to different amounts of white paint while making a custom paint color:

Answers

Answer:

it is 1:5

Step-by-step explanation:

The table shows the amount of red paint that can be added to different amounts of white paint while making a custom paint color:

Paint Color

Quarts of Red Paint Quarts of White Paint

4 20

5 25

7 35

8 40

Which statement is true about the

ratio of red paint to white paint in the paint color?

it is 1:16

it is 16:1

it is 5:1

it is 1:5

Ratio of red paint to white paint in the paint color = 4 : 20

= 4/20

= 1/5

= 1 : 5

Ratio of red paint to white paint in the paint color = 1 : 5

Ratio of red paint to white paint in the paint color = 5 : 25

= 5/25

= 1/5

= 1 : 5

Ratio of red paint to white paint in the paint color = 7 : 35

= 7/35

= 1/5

= 1 : 5

Ratio of red paint to white paint in the paint color = 8 : 40

= 8/40

= 1/5

= 1 : 5

help!!!!!

Describe the graph of the function. y = VX-6 +2

I NEED HELP ASAP​

Answers

Answer:

First answer choice: [tex]y=\sqrt{x}[/tex] shifted right 6 units and down 2 units.

Step-by-step explanation:

Graph

1st choice :) I think

What is the slope of the line represented by the equation y=
iy=-x+ .
0

○ 4
0를
O
0를

Answers

Answer:

-1/2

Step-by-step explanation:

Equation is put in slope intercept form:

y = mx + b

Where m = slope

-1/2 takes the place of m meaning that the slope is -1/2

Using linear function concepts, it is found that the slope of the line is of:

[tex]-\frac{1}{2}[/tex]

What is a linear function?

A linear function is modeled by:

[tex]y = mx + b[/tex]

In which:

m is the slope, which is the rate of change, that is, by how much y changes when x changes by 1.b is the y-intercept, which is the value of y when x = 0.

In this problem, the equation is:

[tex]y = -\frac{1}{2}x + \frac{1}[4}[/tex]

Thus the slope is of [tex]m = -\frac{1}{2}[/tex].

More can be learned about linear function concepts at https://brainly.com/question/24808124

heya kitties. graph y−5=43(x−5)

coordinates plz

Answers

Given:

The equation is:

[tex]y-5=\dfrac{4}{3}(x-5)[/tex]

To find:

The graph of the given equation.

Solution:

Point slope form of a line is:

[tex]y-y_1=m(x-x_1)[/tex]               ...(i)

We have,

[tex]y-5=\dfrac{4}{3}(x-5)[/tex]           ...(ii)

On comparing (i) and (ii), we get

[tex]x_1=5,y_1=5,m=\dfrac{4}{3}[/tex]

It means, slope of the line is [tex]\dfrac{4}{3}[/tex] and it passes through the point (5,5).

Putting x=2 in the given equation, we get

[tex]y-5=\dfrac{4}{3}(2-5)[/tex]

[tex]y=\dfrac{4}{3}(-3)+5[/tex]

[tex]y=-4+5[/tex]

[tex]y=1[/tex]

The line passes through the point (2,1).  Plot the points (2,1) and (5,5) on a coordinate plane and connect them by a free hand curve as shown below.

Mike invested $93,000 at 8.19% compounded weekly.
What will Mike's account balance be in 11 years?

Answers

Answer:

$4920446.6202

Step-by-step explanation:

the formula for compound interest is: P(1+ r/n)^n*t

p= principle (the amount of money you invest/ start with)

r= the interest rate (which is the percentage in decimal)

n= the number of times its compounded per year (52 weeks per year)

t= the amount of time (usually in years)

now, you plug the numbers youre given into the formula:

93000(1+0.0819/52)^52*11

= $4920446.62402

hope this helps :)

Mile's account balance after 11 years will be $228,786.8

What is the formula for compound interest?

"[tex]A=P(1+\frac{r}{n} )^{nt}[/tex]

where A = Accrued amount (principal + interest)

P = Principal amount

r = interest rate as a decimal

R = interest rate as a percent

r = R/100

n = number of compounding periods

t = time in years"

For given question,

P = $93000,

t = 11 years

n = 52 (weekly compounding)

R = 8.19%

So, the interest rate in decimal would be,

[tex]\Rightarrow r =\frac{8.19}{100}\\\\\Rightarrow r =0.0819[/tex]

Using the formula of compound interest,

[tex]\Rightarrow A=P(1+\frac{r}{n} )^{nt}\\\\\Rightarrow A=93000(1+\frac{0.0819}{52} )^{52\times 11}\\\\\Rightarrow A=93000(1+\frac{0.0819}{52} )^572\\\\\Rightarrow A=228,786.8[/tex]

Therefore, Mile's account balance after 11 years will be $228,786.8

Learn more about the compound interest here:

https://brainly.com/question/22979103

#SPJ2

What percentage of temperatures are below 55°? A. 50% B. 25% C. 75% D. 20%

Answers

Step-by-step explanation:

C

im not sure of my answer. hope to help

I think d because others one

which of the following are solutions to the equation 2cos2x-1=0

Answers

Answer:

x = π/4 and 5π/4

Step-by-step explanation:

Given the expression;

2cos²x - 1 = 0

2cos²x = 0+1

2cos²x = 1

cos²x = 1/2

cos x = ±√1/2

cos x = ±0.7071

x = arccos(±0.7071)

x = ±45 degrees

Convert to radians

Sence 180° = π rad

45°  = x

x = 45π/180

x = π/4

If x = -45°

Since cos is negative in the 3rd quadrant

x = 180 + 45

x = 225°

x = 225π/180

x = 45π/36

x = 5π/4

∠1 and ∠2 are supplementary angles. m∠1 = x − 39, and m∠2 = x + 61. Find the measure of each angle.
∠1 = 79, ∠2 = 101
∠1 = 40, ∠2 = 150
∠1 = 40, ∠2 = 140
∠1 = 79, ∠2 = 111

Answers

the third choice is correct.
X-39 + x+61 = 180
2x + 22 =180
2x = 158
X= 79
then you plug into the angles:
79 - 39 = 40 — angle 1
79 + 61 = 140 — angle 2

140 plus 40 is 180.

HELP SOMEONE PLEASE!!!!!!!!!!!!!!!!!!!

Answers

Answer:

3/10

Step-by-step explanation:

The question asks for the probability of finding 4 or more, so first, you must add up all of the teenagers that have 4 or more shoes. Both the 4 and 5 categories fit, so add 20 and 10. Then, find the total number of people in the sample size. Overall, 100 teenagers are represented. Finally, put the number of people that have 4 or more shoes, 30, over the sample size, 100. This equals 30/100 which should be simplified to 3/10.

this is too hard for me lol

Answers

Answer:

<16 should be 70 degrees. Correct me if I'm wrong.

Answer:

70°

Step-by-step explanation:

[tex]m \angle \: 1 + m \angle \: 2 = 110 \degree...(given) \\ \\ m \angle \: 11 + m \angle \:1 2 = m \angle \: 1 + m \angle \: 2 \\ (corresponding \: \angle s) \\ \implies \: m \angle \: 11 + m \angle \:1 2 = 110 \degree \\ \\ now \: \\ m \angle \: 14 + m \angle \:1 5 = m \angle \: 1 1+ m \angle \: 12 \\ (vertical \: \angle s) \\ \implies \: m \angle \: 14 + m \angle \:1 5 = 110 \degree \\ \\ m \angle \: 13 = m \angle \: 16 \: \\ (vertical \: \angle s) \\ \\next \\ m \angle \: 1 1+ m \angle \: 12 + m \angle \: 1 3+ \\ m \angle \: 14 + m \angle \: 1 5+ m \angle \: 16 = 360 \degree \\ \therefore \: (m \angle \: 1 1+ m \angle \: 12 ) + (m \angle \: 1 4+ \\ m \angle \: 15) + m \angle \: 1 3+ m \angle \: 16 = 360 \degree \\ \therefore \:110\degree + 110 \degree + m \angle \: 1 6+ m \angle \: 16 = 360 \degree \\ 220 \degree + 2m \angle \: 16 = 360 \degree \\ 2m \angle \: 16 = 360 \degree - 220 \degree \\ 2m \angle \: 16 = 140 \degree \\ \therefore \: m \angle \: 16 = \frac{140 \degree}{2} \\ \\ \huge \red{ \therefore \: m \angle \: 16 =70 \degree}[/tex]

Find the area in km2. Round answer to the nearest tenth if necessary

Answers

Area of a triangle formula: 1/2 x base x height

Area = 1/2 x 2 x 3.8

Area = 3.8 square km

Answer:

The area of triangle = 3.8km²

Step-by-step explanation:

Given :-

A triangle having length of base is 2km and length of height is 3.8km.

To find :-

Area of triangle.

Solution :-

We know that ,

Area of triangle = ½ × Base × height

Substitute the values.

Area of triangle = ½ × 2km × 3.8km

Multiply 2km by 3.8km.

Area of rectangle = ½ × 7.6km²

Divide by 2 .

Area of triangle = 3.8km²

Let $S$ be the set of points $(a,b)$ in the coordinate plane, where each of $a$ and $b$ may be $-1$, 0, or 1. How many distinct lines pass through at least two members of $S$

Answers

Answer:

20 Lines

Step-by-step explanation:

According to the Question,

Given That, Let S be the set of points (a, b) in the coordinate plane, where each of a and b may be -1, 0, or 1.

Now,  the total pairs of points which can be formed is 9

And, the line passing through 2 such points 9c2 = 9! / (2! x 7!) = 9x4 ⇒ 36

Here, We have overcounted all of the lines which pass through three points.

And, each line that passes through three points will have been counted 3c2 = 3! / 2! ⇒ 3 times

Now, the sides of the square consist of 3 points. We have counted each side thrice, so 4*2 are repeated.

Therefore, the distinct lines pass through at least two members of S is 3 horizontal, 3 vertical, and 2 diagonal lines, so the answer is 36 - 2(3+3+2) = 20 Lines

What is the value of x to the nearest tenth?
A) 9.2
B) 7.2
C) 4.8
D) 12.0

Answers

Answer:

B. 7.2

Step-by-step explanation:

radius=24÷2= 12

other line =19.2÷2=9.6 because line from centre bisects chord and is perpendicular to it

Therefore: X²=√ 12²-9.6² ( theorem of Pythagoras)

X=7.2

What is the measure of x?

Answers

Answer:

Step-by-step explanation:

Use similar triangles and proportions to solve:

[tex]\frac{4}{6}=\frac{10}{6+x}[/tex] and cross multiply:

4(6 + x) = 60 and

24 + 4x = 60 and

4x = 36 so

x = 9

Find the measure of the missing angle using the exterior angle sum theorm.

Answers

Answer:

85°

Step-by-step explanation:

The exterior angle of a triangle is=sum of the opposite interior angles

So

? °=45°+40°

AD = 6, DC = 8, find DB

Answers

Answer:

10

Step-by-step explanation:

use a^2+b^2=c^2

6^2+8^2=c^2

100=c^2

sqrt it and you get 10 for AC and AC=DB

Answer:

according to Pythagoras theorem, c²=a²+b²

|AC|²=|AD|²+|DC|²

|AC|²=6²+8²

|AC|²=36+64

|AC|²=100

√|AC|²=√100

|AC|=10

Graph the solutions of the inequality on a number line. Describe the solution to the inequality. c ≤ -2
open dot at –2; shade all the points to the right of –2
closed dot at –2; shade all the points to the right of –2
open dot at –2; shade all the points to the left of –2
closed dot at –2; shade all the points to the left of –2

Answers

Answer:

The inequality is 2 ≤ x < 8.

A closed dot represents ≤, and an open dot represents <. Since x can represent all values between 2 and 8, you will shade in between 2 and 8 on the number line. x is greater than or equal to 2, so there will be a closed dot on 2. x is less than 8, so there will be an open dot on 8.

The answer is 'number line with a closed dot on 2 and an open dot on 8 and shading in between'.

The solution to the inequality c ≤ -2 is closed dot at –2; shade all the points to the left of –2

What is an inequality?

An inequality is an expression that shows the non equal comparison of two or more numbers and variables.

A number line can be used to represent numbers placed on regular intervals. A number line can be used to represent an inequality.

The solution to the inequality c ≤ -2 is closed dot at –2; shade all the points to the left of –2

Find out more on inequality at: https://brainly.com/question/24372553

Which of the following are true statements? Check all that apply. (WILL GIVE BRAINLIEST)

A. log(M/N) = (logM)/(logN) B. logM+logN=log(MN)
C. log(M/N)=logM-logN
D. logM-logN=log(M-N)

Answers

Answer:

B and C are the correct answers

Step-by-step explanation:

Hope this helps!

The statement B and C are true according to logarithm rules.

What are logarithms?

In mathematics, the logarithm is the inverse function to exponentiation. That means the logarithm of a number x to the base b is the exponent to which b must be raised, to produce x.

A) log(M/N) =log M/log N

Quotient rule is log (x/y)=log x-log y

So, which in not true

B) logM+logN=log(MN)

Product rule is log(xy)=logx+logy

So, the statement is ture

C) log(M/N) =log M - log N

Quotient rule is log (x/y)=log x-log y

So, the statement is ture

D) log M - log N=log(M-N)

Quotient rule is log (x/y)=log x-log y

So, which in not true

Therefore, the statement B and C are true according to logarithm rules.

To learn more about the logarithmic equation visit:

https://brainly.com/question/14406101.

#SPJ2

(Ignore)-Character fill- jsysbssmhzbsskkdhsbsnsdjsvsvsbsjhs

Answers

Step-by-step explanation:

[tex]x = kh \\ x = 33 \\ hj = 4x + 9 \\ x = 9 - 4 \\ x = 5 \\ jk = 3x + 3 \\ x = 3 - 3 \\ x = 0[/tex]

Which function is graphed below?

algebra 2

Answers

Answer:

x=0..............................

It is an exponential decay curve.
The general form of these is y = k.a^x where 0 < a < 1 [negative values for a are much more complicated]
Some key points can be read from the graph - (0, 3) (1, 1) and less accurately (-1, 9) and (2, 1/3)
This points the the equation y = 3(1/3)^x

PLS HELP i just need the answer :,(((((

what is the equation of the quadratic graph with a focus of (5, −1) and a directrix of y = 1?

Answers

Answer:

check the picture below.

so the focus point is at 5, -1 and the directrix is above it, meaning is a vertical parabola, and is opening downwards, like in the picture.

keep in mind that the vertex is half-way between those two fellows, at a "p" distance from either, in this case 1 unit, since the parabola is opening downwards, "p" is negative then, or -1, and the vertex will be from 5, -1 up one unit, so 5,0.

Find the area of the rhombus

Answers

Answer:

The rhombus has an area of 240 units. (2nd one on the left).

Step-by-step explanation:

The intersection of diagonals meet at right angles. That means that the labeled sides are the hypotenuse and 1 leg of a right angle triangle. You need to find the other leg by using the Pythagorean Theorem

a^2 + b^2 = c^2

a = 8

b = ?

c = 17

8^2 + b^2 = 17^2

64 + b^2 = 289                 Subtract 64

b^2 = 289-64

b^2 = 225                         Take the square root of both sides.

sqrt(b^) = sqrt(225)

b = 15

Find the area of 1 triangle

Area = 1/2 * a * b

a = 8

b = 15

Area = 1/2 * 8 * 15

Area = 60

Now find the area of 4 triangles.

4*60 = 240

A particle is projected with a velocity of [tex]29.4ms^-^1[/tex] . Find it's maximum range on a horizontal plane through the point of projection.
A.88.2m B.44.1m C.32.6m D.29.4m E.14.7m

Answers

A.88.2m

Answer:

Solution given:

initial velocity[u]=29.4m/s

g=9.8m/s²

maximum range=?

now

we have

[tex]\theta=90°[/tex]

maximum range =[tex]\frac{29.4²*sin90}{9.8}=88.2m[/tex]

The initial velocity is,

→ u = 29.4 m/s

General assumption,

→ g = 9.8m/s²

→ θ = 90°

Then the maximum range is,

→ (29.4² × sin90)/9.8

→ 88.2 m

Hence, option (A) is answer.

Please help! will give brainly

Answers

Answer:

5/6

Step-by-step explanation:

first form numerator you will multiply by π it means 3.14 multiply 11

the answer equals 34.54 ..and then you will use √ it means square root....so the answer is 5

the annwer is

5/6

Answer:

Step-by-step explanation:

[tex]\frac{11\pi }{6}[/tex] =  11 × [tex]\frac{\pi }{6}[/tex] = (12 - 1) × [tex]\frac{\pi }{6}[/tex]

The terminal point located in IV quarter ⇒ x-coordinate is positive and y-coordinate is negative.

(x, y) = ( [tex]\frac{\sqrt{3} }{2}[/tex] , - [tex]\frac{1}{2}[/tex] )

please help it's due i will give brainly

Answers

Answer: 1) You forgot to change the sign over there. You have to subtract over there.

2) Just be careful while solving equations while changing the signs

Find the Value of x that makes A parallel to B.

Answers

Answer:

x=60

Step-by-step explanation:

Other Questions
3 things that come to mind when you hear the word drugs d. Application software are developed by software companies 3/4 of the households in a rural area have pets. how many households have pets in this area if there are 1500 total households What are the measures of Angles a,b, and c? show your work and explain your answers. a majority of land in the middle east is unsuitable for crops because what is the value of g In parallelogram QRST if RU=17 find UT. Need the value of P please Find the volume of this squarebased pyramid.10 cm8 cm8 cmV = [?] cm3 Graph the function g(x) = 3^x + 3 and give its domain and range using interval notation. How does adding oxygen (O) to this reaction change the equilibrium?2s02(g) + O2(g) = 2s03(8) A prism and two nets are shown below: Prism 1 E 3 Net A Net Part A: Which is the correct net for the prism? Explain your answer. (2 points) Part B: Write the measurements of Sides AB. BC, and CD of the correct net. (4 points) Part C: What is the surface area of the prism? Show your work. (4 points) Another question pls have a explaination if you know u are correct define one standard kg what are the two region of growing demand of water? Whats the monster x man duo called/referred to when it comes to art bases? I want get cute ideas to draw 2 of my characters in from bases but I can't seem to find any ones that fit the man x monster duo. can someone pls help me i really really need it =3 Write synonymsI haven't gone to the gym since May== 3(4p - 2) +3 = 2p - 5no solution, one solution or infinitely many solutions? The author of passage 1 would probably respond to the reference in passage 2 to the social isolation inherent in living aloneby